4
$\begingroup$

I have already asked this at math.stackexchange, but since no one answered there after my edit, I decided to try here, although it might be a non-research level question.

The following version of Nakayama's lemma is from Matsumura's Commutative Ring Theory:

Let $M$ be a finitely generated $A$-module, $I\subseteq A$ an ideal s.t. $IM=M$. Then there exists an $a\in A$ with $a\equiv 1\pmod{I}$ and such that $aM=0$. In particular, if $I\subseteq\operatorname{rad}(A)$ we have $M=0$.

After the proof of this via a generalized Cayley-Hamilton, he mentions that the result 'can easily be proved [..] by induction on the number of generators of $M$.' I wonder: how? I tried doing it similarly to the inductive proof of the 'in particular' part, but it didn't work out for me (see MSE for more information on what I think I was doing wrong).

Wouldn't I need to be able to find an $N\subseteq M$, $IN=N$, with fewer generators than $M$ in a somewhat obvious way to use the induction hypothesis? How could I do this?

Thanks in advance!

$\endgroup$
4
  • $\begingroup$ You need to do it the other way around. Let $N$ be generated by the first generator of $M$, then $M/N$ needs fewer generators than $M$ and satisfies $I.(M/N)=M/N$, and you can take it from there. $\endgroup$ May 21, 2012 at 8:20
  • 1
    $\begingroup$ @Neil: What should we do with some $a \equiv 1$ mod $I$ satisfying $a M \subseteq N$? @Rand: I think that Matsumura only refers to the particular case that $I \subseteq jac(R)$, this can be done by induction. The general form is equivalent to Cayley-Hamilton and no reasonable induction is possible. $\endgroup$ May 21, 2012 at 8:37
  • 1
    $\begingroup$ I believe Martin is right and said so here: mathoverflow.net/questions/41836/… $\endgroup$ May 21, 2012 at 22:09
  • $\begingroup$ Hey InvisiblePanda, I saw your name in chat.SE and couldn't resist contacting you! I think we have a book series in common :-) The name of this room is pretty funny. $\endgroup$ Dec 29, 2014 at 21:22

1 Answer 1

3
$\begingroup$

After a little work I think I found an inductive proof:

Let $M$ be finitely generated by $n$ elements. We perform induction on $n$. If $n=1$, $M=\langle m\rangle$ and there is some $x\in I$ such that $xm=m$. Thus $1-x$ annihilates $M$.

Assume by induction that we proved the claim for some $n-1$ and suppose $$M = \langle m_1,\ldots,m_n \rangle.$$ Consider $N = M / \langle m_n \rangle$. It is generated by $n-1$ elements and satisfies $N=IN$, so there exists $x \in I$ such that $1+x$ annihilates $N$. Therefore $$(1+x)M \subset \langle m_n \rangle.$$ Since $m_n \in IM$, $$(1+x)m_n \in (1+x)IM = I(1+x)M \subset I\langle m_n \rangle.$$ Take $y\in I$ such that $(1+x)m_n = ym_n$. Then $(1+x-y)m_n = 0$ and $$(1+x)(1+x-y) \in 1+I$$ annihilates all $M$.


In fact, one can use the same method to replace the determinant trick entirely. Unfortunately the resulting polynomial has high degree.

Claim: Let $M$ be finitely generated and $\varphi:M \rightarrow M$ an $R$-module homomorphism satisfying $\varphi(M) \subset IM$ where $I$ is an ideal (possibly all $R$). Then $$\sum_{i=0}^{k-1} a_i \varphi^i + \varphi^k = 0$$ on $M$ for some $a_0,\ldots,a_{k-1} \in I$.

Proof: This is really the same as the previous proof, just replacing $\mathrm{id}_M$ by $\varphi$ in some places and keeping track of what this entails.

The case $M=\langle m \rangle$ is clear: $\varphi(m) = xm$ for some $x \in I$, so $\varphi - x = 0$ on $M$.

Assume by induction that we proved the claim for some $n-1$ and suppose $$M = \langle m_1,\ldots,m_n \rangle.$$ Consider $N = M / \langle \{\varphi^i (m_n)\}_{i\ge 0} \rangle$. It is generated by $n-1$ elements and satisfies $\varphi(N) \subset IN$. Hence there exists some polynomial $p(x)$ (monic, with all but the top coefficient in $I$) satisfying $p(\varphi) = 0$ on $N$. Therefore $$p(\varphi)(M) \subset \langle \{\varphi^i (m_n)\}_{ i\ge 0} \rangle.$$

In fact it suffices to take $$ \langle \{\varphi^i (m_n)\}_{0\le i \le k} \rangle,$$ where $k$ is some natural number (take a maximum over the powers needed for the elements $p(\varphi)(m_i),\ \ i \le n$.)

Since $\varphi^{k+1}(m_n) \in IM$, $$(p(\varphi))(\varphi^{k+1}(m_n)) \in p(\varphi)(IM) = I\cdot p(\varphi)(M) \subset I \langle \{\varphi^i (m_n)\}_{0\le i \le k} \rangle.$$

Take $y_0,\ldots,y_k \in I$ such that $q(\varphi) = \sum_{i=0}^k y_i \varphi^i$ satisfies $$p(\varphi)\circ\varphi^{k+1}(m_n) = q(\varphi)(m_n),$$ and then since $p(\varphi)\circ\varphi^{k+1} - q(\varphi)$ is monic with all but the top coefficient in $I$, $$p(\varphi)\circ(p(\varphi)\circ\varphi^{k+1} - q(\varphi))$$ gives a polynomial of the required form: composition of polynomials of an endomorphism = multiplication of polynomials. So the induction step gives $$(p(z)^2 \cdot z^{k+1} - q(z)p(z))\restriction_{z=\varphi}$$ where $y \in I.$

(edit: thanks to Yakov Varshavsky for pointing out an error near the last step, where I mistakenly thought $q$ could be taken to be a single monomial.)

$\endgroup$
1
  • $\begingroup$ @Martin-Brandenburg, others: (In response to a comment on the question above, which I don't have the reputation to reply to directly.) It's worth noting that Cayley-Hamilton itself is an elementary computation, so it should not be too surprising that a simpler computation is possible for this use-case (Cayley-Hamilton has significant advantages in general, like a degree bound and an explicit description of the polynomial.) This proof is mainly useful for showing determinants are not somehow necessary for the basic theory, e.g. a book like A-M could have been written entirely without them. $\endgroup$ Nov 10, 2019 at 8:48

Your Answer

By clicking “Post Your Answer”, you agree to our terms of service and acknowledge you have read our privacy policy.

Not the answer you're looking for? Browse other questions tagged or ask your own question.